viernes, 28 de noviembre de 2008

Unidad IV........Ley Cero de la Termodinamica

1.- ¿Que propiedades dependen de la temperatura?
R:
1.- Punto de Fusion.
2.- Punto de Ebullicion.
3.- Densidad.

2.- ¿A que se le llama Equilibrio Tèrmico?

R: Se le llama equilibrio termico a a dos sistemas que estan en contacto mecanico directo o separados mediante una superficie que permite la trasnferencia de calor. tambien se dice que los cuerpos se encuentran en equilibrio termico cuando no existe flujo de calor de uno haca otro.

3.- ¿Què es un Aislante Ideal?
R: Es cualquier material que impide la transmision de energia en cualquiera de sus formas, con masa que impide la transportacion de energia; como:
1.- Aislante Termico.
2.- Aislante Elèctrico.

3.- Aislante Acùstico.
4.- Aislante de Microondas.
5.- Aislante de Barreras.

4.- Dibujar un sistema que representa la Ley Cero de la Termodinàmica, iniciando el equilibrio tèrmico.














5.- ¿Cuàndo se dice que dos sistemas estan en equilibrio tèrmico?

R: Cuando ambos cuerpos estan en contacto tèrmico y no existe flujo de calor de uno hacia otro. Ademas las propiedades fisicas del sistema varian con respecto a la temperatura y cambian con el tiempo.


6.- ¿Por què cuando una enfermera toma la temperatura de un paciente, espera que la temperatura del termometro empieza a cambiar?

R: Porque la temperatura del cuerpo varia, segun el

medio; la temperatura interna o cenrtal del cuerpo es regulada de forma precisa y se conserva dentro de limites muy estrechos. Es por eso que la enfermera espera a que la temperatura del termometro deje de cambiar para poder lograr un equilibrio tèrmico entre el paciente y el ambiente.


7.- Mencione tres tipos de dispositivos que miden la temperatura:

R:

1.-Termometro (medicion de temperatura del cuerpo humano o ambiental).

2.-Termopar (medicion de la diferencia de temperatura

entre extremos; denominados punto caliente y punto frio).

3.- Pirometro (mide la temperatura de una sustancia sin necesidad de estar en contacto con ella).


8.- ¿Cual es la temperatura de congelacion del agua en ºF?

R: 32 ºF


9.- Calcular la temperatura Fahrenheit del planeta Venus, si en ºC corresponde a 460.

R: Grados Celsius a Fahrenheit

ºF = ( 1.8 ) ( ºC ) + 32

( 1.8 ) ( 460 ) + 32

460 ºC = 860 ºF


10.- Encontrar la temperatura en la que coinciden las escalas Fahrenheit y Celsius.

R: Ambas coinciden a 40º bajo cero.

11.- La temperatura de la corona solar es de 2 x 10`7 ºC, y la temperatura a la que helio se licua a presion estandar es de 268.93 ºC.

a) Expresar estas temperaturas en ºK.

b) Explica porque suele usarse la escala Kelvin.

R:

a) Grados Celsius a Grados Kelvin

ºK = ºC + 273.15

20000000 + 273.15 = 20000273.15

268.93 + 273.15 = 542.08

b) Solamente se utiliza en experimentos especiales cientificos; el ºK es la unidad de temperatura creada sobre la base del ºC, estableciendo el punto cero, en cero absoluto (-273.15 ºC) y conservando la misma dimension.


12.- Dos vasos de agua, "A" y "B" estan inicialmente a la misma temperatura. La temperatura del vaso "A" aumenta 10 ºF y la del vaso "B" 10 ºK. ¿Cuàl vaso esta ahora a mayor temperatura?
R:
Grados Fahrenheit a Grados Kelvin
ºK = ( ºF + 459.67 ) / 1.8
( 10 + 459.67 ) / 1.8

260.93 ºK
Por lo tanto el vaso "A" tiene ahora mayor temperatura. ( vaso "A"=260.93 ºK y vaso "B"=10ºK)

jueves, 27 de noviembre de 2008

Unidad 4

13.6 Momento de Rotación Magnético sobre un Lazo
Objetivo
El objetivo de esta actividad es desarrollar un entendimiento del momento de rotación sobre un lazo que lleva corriente en la presencia de un campo externo magnético.

Pregunta 1: La Fuerza magnética sobre la Cima del Lazo Si un campo magnético es aplicado en la dirección de x positiva, va a la fuerza magnética sobre el cable ,¿que comprende la cima del lazo ser dirigida en el +x,-x, +y,-y, +z, o la dirección-z?
R: Para determinar la dirección de la fuerza magnética sobre un cable que lleva corriente, imagínese señalar los dedos de su mano derecha en la dirección de la corriente. Rice sus dedos hasta que ellos señalen en la dirección del campo magnético. Su pulgar señala en la dirección de la fuerza magnética sobre el cable.

Pregunta 2: La inversión del Campo Si la dirección del campo magnético es invertida, ¿qué pasará a la dirección de la fuerza magnética sobre cada uno de los cuatro lados del lazo?
R: Si el campo magnetico se incrementa positivamente la fuerza magnetica actúa para los lado exteriores del campo donde se encuentra, en cambio si se incremente el campo magnetico negativamente, las fuerzas en z actuan hacia el exterior y las fuerzas en y actúan hacia el interior, en cambuo el campo magnético ca al medio de la figura, hacia adentro.

Pregunta 3: La Fuerza neta sobre el Lazo
¿Qué interpreta la fuerza neta sobre el lazo?
R: Las fuerzas en la parte superior e inferior del bucle son iguales en magnitud (porque de igual magnitud actual, la duración y el campo magnético), pero en dirección opuesta, porque de lo contrario las direcciones de las corrientes. Por lo tanto, estas dos fuerzas de suma a cero. Lo mismo es cierto para las fuerzas de las dos partes en el cable de bucle. Por lo tanto, la fuerza neta en el bucle es igual a cero.

Pregunta 4: Fuerzas en el giro del lazo:
¿Qué pasará a la magnitud y la dirección de las fuerzas magnéticas que actúan sobre los cuatro segmentos de cable si el lazo es hecho girar a un pequeño ángulo positivo?

R: Simplemente la magnitud empieza a tomar valores negativos ya que se mueva hacia abajo y las fuerzas se mueven junto con el lazo y no aumentan. (10º)

Pregu
nta 5: Giro del Lazo a 90 °
¿Qué pasará a la magnitud de la fuerza sobre el cable superior cuando el lazo es hecho girar a +90 °?
Compruebe su respuesta por haciendo girar el lazo a +90 °.
R: se podria decir que obtiene un valor de cero, debido a que no se encuentra ni en direccion hacia arriba ni hacia abajo; simplemente queda indicada en el lazo.

Pregunta 6: Giro del Lazo, más de 90 °

¿Qué pas
ará a la magnitud y la dirección de la fuerza sobre el cable superior cuando el lazo es hecho girar más de +90 °?
R: La magnitud seguira con valores negativos y la direccion tomara una direccion hacia abajo en el eje de las "y"; el angulo que se le da es de 110º

Pregunta 7: Giro del Lazo:
¿Puede la fuerza neta de la red forzada sobre el lazo, ser alguna vez no nula?
Compruebe su respuesta por ajustando el área del lazo, el campo aplicado, y/o la orientación angular.


Pregunta 8: Giro del Lazo, El Momento de Rotación
¿Cuál será la dirección del momento de rotación neto sobre el lazo si es hecho girar a un pequeño ángulo positivo?
Compruebe su respuesta por mostrando el momento de rotación y haciendo girar el lazo.

Pregunta 9: Giro del Lazo; Angulos Negativos.
¿Cuál será la dirección del momento de rotación neto sobre el lazo si es hecho girar a un pequeño ángulo n
egativo?
Compruebe su respuesta por haciendo girar el lazo.


Pregunta 10:
¿El Momento de rotación Máximo Positivo con el campo magnético que señala en la dirección +x, para lo que el ángulo de orientación experimentará el lazo el momento de rotación máximo positivo?
Compruebe su respuesta por haciendo girar el lazo.


Pregunta 11:
¿La dependencia del area hace que el tamaño del momento de rotación depende del área del lazo? ¿Si es así, cómo? Compruebe su predicción por variando el tamaño del lazo.
R: Si la
anchura del lazo es aumentada, entonces el brazo de palanca de las fuerzas sobre los aumentos de segmentos verticales de cable, conduciendo a un momento de rotación mayor. Si la altura del lazo es aumentada, entonces la longitud de los aumentos de segmentos verticales de cable, conduciendo a una fuerza mayor, que conduce a un momento de rotación mayor. ¡Así, aumentando las dimensiones del lazo aumenta el momento de rotación neto sobre el lazo!




Pregunta 12: La Regla General
Por diversas orientaciones, examinar de cerca la relación entre el campo magnético, momento dipolo magnético, y el par de vectores. ¿Puedes pensar en una simple relación vector que resume la dependencia de la torsión en el momento de dipolo magnético y el campo magnético?
R: El par máximo es cuando los otros dos vectores son 90 ° aparte, y es igual a cero cuando los otros dos vectores son o bien 0 ° ó 180 ° de separación. Esto suena un poco como una función de seno. El vector entre productos,


,tiene un
a magnitud que depende del ángulo entre y en una forma sinusoidal. De hecho,



especifica correctamente la magnitud y la dirección de la torsión en una corriente portadora de bucle.

Unidad 3....Campo Magnetico Problemas

1.- Una bobina recatangular de 50 vueltas y dimensiones de 5 x 10 cm. se deja caer desde una posicion donde B=0 hasta una nueva posicion donde B=0.5 T. y se dirige perpendicularmente al plano de la bobina, calcule la magnitud de la fem. promedio inducida en la bobina si el desplazamiento ocurre en 0.250 s.
Datos:
N= 50
a= 5cm

b= 10cm

B1= 0

B2= 0.5
T
t= 0.250 s
fem=?

2.- Una bobina circular de alambre de 25 vueltas tiene un diametro de 1 cm. La bobina se coloca con su eje a lo largo de la direccion del campo magnetico de la tierra de 50 uT, y luego en 0.2s. Se gira a 180 grados. Cual es la fem promedio generada en la bobina.




3.- Un anillo de aluminio con un radio de 5 cm y una resistencia de 3 x10-4 ohms, se coloca sobre la parte superior de un largo solenoide con un nucleo de aire, 1000 vueltas por metro y un radio de 3cm. Suponga que el solenoide produce un campo despreciable afuera de su area de seccion transversal.

a) si la corriente en el solenoide esta aumentando a razon de 270 A/s. ¿ cual es la corriente inducida en el anillo?
b) en el centro del anillo ¿ cual es el campo magnetico producido po la corriente inducida en el anillo?
c) ¿ cual es la direccion de este campo?



4.- Encuentra la corriente que atraviesa la seccion PQ la cual tiene una longitud alfa= 65cm. El circuito se localiza en un campo magnetico cuya magnitud varia con el tiempo de acuerdo con la expresion B= ( 1 x 10 -3 T/s) t. Suponga que la resistencia por longitud del alambre es 0.1 homs/m.


5.- Una bobina que se enrolla con 50 vueltas de alambre en la forma de un cuadrado se coloca en un campo magnetico de modo que la normal al plano de la bobina forme un angulo de 30 grados con la direccion del campo cuando el campo magnetico se incrementa uniformemente de 200 uT a 600uT. En 0.4 s. Una fem de 80 mv de magnitud se induce en la bobina ¿cual es la longitud total del alambre?





6.-Una bobina circular que encierra una area de 100m2 esta integrada por 200 vueltas de alambre de cobre, al principio un campo magnetico uniforme de 1.10 T. apunta perpendicularmente hacia arriba a traves del plano de la bobina. la direccion del campo se invierte despues durante el tiempo que el campo esta cambiando su direccion. ¿ cuanta carga fluye a traves de la bobina si R= 5 ohms?





7.- Una bobina rectangular con resistencia R tiene N vueltas, cada una de longitud L y ancho w, la bobina se mueve dentro de un campo magnetico uniforme B a velocidad V ¿ cuales son la magnitud y direccion de la fuerza resultante sobre la bobina?
a) cuando esta entra al campo magnetico.
b) cuando se mueve dentro del campo magnetico.
c) cuando sale del campo.





8.- Dos rieles que tienen resistencia despreciables estan separados a 10 cm por medio de un resitor de 5 homs. el circuito contiene tambien dos barras metalicas con resistencia de 10 homs y 15 homs que se desliza a lo largo de los rieles. Las barras se alejan del resitor con rapidez constante de 4m/s y 2 m/s respectivamente. se aplica un campo magnetico uniforme de 0.01 T de magnitud perpendicular al plano de los rieles, determine la corriente en el resistor de 5 ohms.




9.- Una bobina de 0.100m2 de area esta girando a 60 rev/s con el eje de rotacion perpendicular a un campo magnetico de 0.200 T.
a) si hay 1000 vueltas en la bobina ¿cual es el maximo voltaje inducido en el ?
b) cuando el maximo voltaje ocurre ¿ cual es la orientacion de la bobina repecto del campo magnetico?



10.- Un largo solenoide, cuyo eje coincide con el eje x consta de 200 vueltas por metro de alambre que conduce una corriente estable de 15A. se forma una bobina enrollando 30 vueltas de lambre delgado alrededor de un armazon circular que tiene un radio de 8 cm. la bobina se supone dentro del solenoide y se monta sobre un eje que esta a un diametro de la bobina y coincide con el eje y, despues de la bobina se hace girar con una rapidez angular de 4 (3.1416) rad/s. el plano de la bobina esta en el plano yz en t=0. Determine la fem desarrollada en la bobina como funcion del tiempo.




11.- En el ecuador cerca de la superficie de la tierra, el campo magnetico es aproximadamente de 50 homs/T. con direccion norte y el campo electrico es cercano a 100 N/C hacia abajo en clima favorable encuentre la fuerza gravitacional, electrica y magentica sobre un electron que se mueve a una velocidad instantanea de 6 x 10 6 m/s en direccion este en dicho ambiente.



12.- Un alambre conduce una corriente estable de 2.40 A una seccion recta del alambre mide 0.750 m de largo y se encuentra a lo largo del eje x dentro de un campo magnetico uniforme de magnitud b= 160 T en la direccion x positiva si la corriente esta en la direccion +x ¿ cual es la fuerxa magnetica sobre la seccion positiva del alambre?


13.-Una corriente de 1.70 mA se mantiene en una espira de circuito individual de 2 m de circunferencia un campo magnetico de 0.8 T se dirige paralelo al plano de la espira.
a) calcule el momento magnetico de la espira.
b) ¿cual es el momento de torsion ejercido sobre la espira por el campo magnetico?


14.- Un alambre de 40 cm de largo conduce una corriente de 20 A se dobla en una espira y se coloca con su normal perperdicular a un campo magnetico con una intensidad de 0.520 T. ¿ cual es el momento de torsion sobre la espira si se dobla en la forma de:
a) un triangulo equilatero
b) un cudrado
c) un circulo
d) cual momento de torsion es mas grande?



15.- Un ion positivo con una sola carga tiene una masa de 3.20 x 10 -26 kg. despues de que es acelerado desde el reposo a traves de una diferencia de potencial de 833 v. el ion entra en un campo magnetico de 0.920 T. a lo largo de una direccion perpendicular a la direccion del campo. Calcule el radio de la trayectoria del ion en el campo.









jueves, 23 de octubre de 2008

Unidad 3....Campo Magnetico

Campo Magnetico de un Alambre


Pregunta 1: La Dirección del Campo Magnético
¿Cuál será el campo magnético aspecto positivo cuando las corrientes actuales a través del alambre? (Positivos actual se define a fluir fuera de la pantalla.)
Aumentar la corriente en el cable y comprobar su predicción.
R:Al mover las flechas azules del simulador que son las encargadas de indicar la carga y líneas de campo magnético del alambre; se observa que al traspasar una corriente positiva, las líneas de dicho campo aumentan conforme aumentamos los valores de la corriente y por consiguiente también el flujo aumenta. Pregunta 2: Orientación del Campo Magnético
¿Qué hace el ángulo de campo magnético que en relación con la posición del vector que conecta el cable hasta el punto de interés?

R: Cuando se cambia la dirección del vector del campo magnético en un punto especifico se puede ver que las líneas del campo permanecen igual, sin embargo la distancia desde el punto de origen hasta un punto especifico varia dependiendo de su ubicación y es representado por la letra “r (radio)”, además también se genera un cambio radical en la intensidad de campo magnético representado por la letra B.


Pregunta 3: Magnitud a lo largo de una Línea Radial
¿La magnitud de los cambios sobre el terreno a lo largo de una línea que se extiende radialmente lejos de los cables?
Cuidadosamente arrastre el vector del campo magnético sobre una recta, radial fuera de la línea del cable para comprobar su respuesta.


R: Al parecer la magnitud del campo magnético no cambia ya que solo se esta ocasionando un movimiento en el vector director y lo único que cambia es el radio y la intensidad del campo.





Pregunta 4: Magnitud de Campo a lo largo de una Línea
¿La magnitud de los cambios sobre el terreno a lo largo de las líneas de campo circular?
Cuidadosamente arrastre el campo magnético en torno a un vector de la circular líneas de campo magnético para comprobar su respuesta.

R: De la misma forma que en la respuesta anterior, no se observa un cambio en la magnitud del campo al arrastrar el vector director alrededor de las líneas circulares. Lo único que cambias es el radio en el vector director y la intensidad de campo.


Pregunta 5: La Dependencia de la Actual
¿Qué va a pasar con la magnitud y la dirección del campo magnético, en el punto en el espacio que están estudiando, si el actual es el aumento?
Aumentar la actual y comprobar su predicción.



R: Al momento de realizar el experimento se observa de forma gradual que tanto la magnitud como la dirección del campo magnético varían sustancialmente al generar un aumento en la corriente.





Pregunta 6: Flipping la Actual
¿Qué va a pasar con la magnitud y la dirección del campo magnético, en el punto en el espacio que están estudiando, si la corriente es volteado de positivo a negativo?
Voltear la corriente en el cable y comprobar su predicción. predicción.

R: En este punto se observa que las circunferencias verdes que indican la magnitud del campo magnético cambian el sentido inverso de dirección, es decir, de derecha a izquierda, además también se produce un cambio en el vector director del campo magnético en dirección hacia abajo.






Pregunta 7: Plan Limaduras de Hierro
¿Qué va a pasar con el patrón de limaduras de hierro, si la corriente es volteada a un valor positivo?

Voltear la corriente en el alambre de vuelta a un valor positivo y comprobar su predicción.

R: En este caso la dirección de la flecha con respecto a las circunferencias verdes que indican el campo magnético no cambian de dirección, sin embargo el radio y la intensidad de la magnitud toman valores distintos, además el vector director regresa a la posición
anterior, es decir a su posición original.




Pregunta 8: Dependencia Funcional en la Actual
¿Cuál es la dependencia funcional de campo magnético sobre las actuales para un recto, actual portadora de alambre?

R: Debido a que el campo magnético aumenta en una cantidad constante, para un aumento constante en la corriente, la dependencia debe ser lineal. Desde las gotas del campo a 0 cuando la corriente es 0, después la dependencia linear debe ser, de hecho, proporcional.




Pregunta 9: La Dependencia Funcional en la Distancia
¿Cuál es la dependencia funcional de campo magnético sobre una distancia de la recta, actual portadora de alambre?


R:Debido a que el campo magnético disminuye por un factor de dos ,cuando los aumentos de la distancia por un factor de dos la dependencia deben ser e l ~ 1/r. de B.





Pregunta 10: Ley Biot-Savart
¿Cuál es la distancia de un cable de llevar más allá de 10 A que el campo magnético es menos de 15 μT?
Utilizar la simulación para comprobar su respuesta.

Pregunta 11: Rompecabezas Biot-Savart
A 2 cm de largo objeto se coloca en el campo magnético de un alambre de 15 A. Un extremo del objeto está expuesto a un campo de 35 μT. ¿Qué gama de campos magnéticos de mayo, el otro extremo del objeto a ser expuestos?
Utilizar la simulación para comprobar su respuesta.

R: El extremo del objeto en el campo de 35 µT se debe localizar en:

B = µI/2πr

r = µI/2πB

r = (4π x 10-7 Tm/A) (15 A)/2π (35 x T) 10-6

r = 0.086 M.

Así, el otro extremo del objeto debe estar entre 10.6 cm y 6.6 cm del alambre. Tapar esta dos distancias en la ley de Biot-Savart rinde campos magnéticos el µT 28 del µT y 45, respectivamente.




sábado, 18 de octubre de 2008

Cuestionarios

Cuestionario Equipo 1

Fuerza Electrostática, Ley de Coulomb.

1.- Dos esferas metálicas cuelgan de hilos de nylon. Cuando se colocan próximas entre si tienden a atraerse. Con base sólo en esta información, analice los modos posibles en que podrían estar cargadas las esferas. Es posible que, luego de tocarse, las esferas permanezcan adheridas una a la otra? Explicar la respuesta.

R: Las esferas al decir que se atraen deben de estar cargadas con signos contrarios es decir una de ellas ha de ser de signo positivo y la otra de signo negativo. Lo último depende de la magnitud de la fuerza de atracción entre ambas. Así como del valor de las cargas, pero si la fuerza eléctrica de atracción es suficiente, puede ser que después de tocarse, permanezcan adheridas.

2.- Los buenos conductores elçtricos, como los metales, son típicamente buenos conductores del calor; los aisladores eléctricos, como la madera, son típicamente malos conductores del calor. Explicar por qué tendría que haber una relación entre la conducción eléctrica y la conducción térmica en estos materiales.

R: Existe una relación por que en el caso de los metales, los electrones que se encuentran en la capa de valencia están libres; por lo cual tienen facilidad para el intercambio de estos mismos, este intercambio permite tanto la conductividad eléctrica así como de energía termina debido al constante movimiento que hay entre los electrones que a su vez producen calor.

3.- Tres cargas puntuales están dispuestas en línea . La carga Q3 = + 5 nC está en el origen. La carga Q2 = - 3 nC está en x = 4 cm. La carga Q1 = está en x = + 2 cm. Cuál es la magnitud y el signo de Q1 , si la fuerza neta sobre Q3 es cero?

R:

F_3_/_2= ( 9 X 10^9 Nm^2/C^2)(5 X 10^9C)(-3 X 10^9C)/(0.4m)^2
F_3_/_2 = 8.4375 X  10^7 N

8.4375 X  10^7 N = (9 X 10^9 Nm^2/C^2)(5 X 10^9C)(q_1)/(0.2 m)^2
(8.4375 X  10^7 N)(0.2 m)^2= (9 X 10^9 Nm^2/C^2)(5 X 10^9C)(q_1)

(8.4375 X  10^7 N)(0.2 m)^2/(9 X 10^9 Nm^2/C^2)(5 X 10^9C) = (q_1)

(q_1) = 7.5 X 10^9 C = 7.5 nC

comprobacion

F_3_/_1 = (9 X 10^9 Nm^2/C^2)(7.5 X 10^9 C)(5 X 10^9C)/(0.2 m)^2

F_3_/_1 = 8.4375 X  10^7 N

F_3_/_1 - F_3_/_2 = 8.4375 X  10^7 N - 8.4375 X  10^7 N = 0

4.- Se coloca una carga puntual de 3.5 uC, a 0.8 m a la izquierda de una segunda carga puntual idéntica. Cuáles son las magnitudes y direcciones de las fuerzas que cada carga ejerce sobre la otra?

R:

F_1_/_2 = (9 X 10^9 Nm^2/C^2)(3.5 X 10^-^6 C)(3.5 X 10^-^6 C)/(0.8 m)^2
F_1_/_2 = 0.1722



Cuestionario Equipo 2

Fuerza Electrostática, Principio de Superposición.

1.- Dos cargas puntuales iguales ejercen fueras iguales una sobre la otra. Pero si una carga es el doble de la otra, siguen ejerciendo fuerzas iguales una sobre la otra, o una ejerce dos veces más fuerza que la otra?

R: Son iguales ya que el aumentar una de las cargas la fuerza de atracción y/o repulsión entre ellas se mantienen constante por parte de ambas.

2.- Qué semejanza presentan las fuerzas eléctricas con las fuerzas gravitatorias? Cuáles son las diferencias más significativas?

R: La semejanza es que ambas se basan en la atracción entre dos o más cuerpo distintos. Y la diferencias mas significativas, es que la fuerza gravitatoria se basa conforme a la masa de los cuerpos mientras que la fuerza eléctrica es necesario que las partículas o cuerpos estén cargados ya será positivamente o negativamente.

3.- A dos esferas pequeñas de plástico se les proporciona una carga eléctrica positiva. Cuando están a 15 cm de distancia una de la otra, la fuerza de repulsión entre ellas tiene una magnitud de 0.22 N. Qué carga tiene cada esfera, a) Si las dos cargas son iguales?, b) Si una esfera tiene cuatro veces más carga que la otra?
R:
A) si las dos cargas son iguales

0.22= (9 X 10^9 Nm^2/C^2)(q_1)(q_2)/(0.15 m)^2

(0.22) (0.15 m)^2= (9 X 10^9 Nm^2/C^2)(q_1)(q_2)

(0.22) (0.15 m)^2/ (9 X 10^9 Nm^2/C^2) = (q_1)(q_2)

(q_1)(q_2)= \sqrt[2]{5.5 X 10^-^13} (q_1)(q_2) = 7.4161 X 10^-^7


F_1_/_2 = (9 X 10^9 Nm^2/C^2)(7.4161 X 10^-^7)(7.4161 X 10^-^7)/ (0.15 m)^2

F_1_/_2 = 0.22 N

B) si una esfera tiene 4 veces mas carga que la otra

q_1q_2 = 5.5 X 10^-^13

4x(x) = 5.5 X 10^-^13

4X^2= \sqrt[2]{5.5 X 10^-^13}

X= \sqrt[2]{5.5 X 10^-^13}/4
X= 3.7080 X 10^-^7

4(3.7080 X 10^-^7)(3.7080 X 10^-^7)

q_1q_2 = 5.5 X 10^-^13

F_1_/_2 = (9 X 10^9 Nm^2/C^2)(1.4832 X 10^-^6)(3.7080 X 10^-^7)/(0.15 m)^2
F_1_/_2 = 0.22N

4.- Tres cargas puntuales están ordenadas a lo largo del eje de las “x”. La carga Q1= +3 uC está en el origen, y la carga Q2 = - 5 uC está en x = 0.2 m. La carga Q3 = - 8 uC . Dónde esta situada Q3 si la fuerza neta sobre Q1 es 7 N en la dirección – x?
R:

Q_1 = F_1_/_2 - F_2_/_3

F_1_/_2 = (9 X 10^9 Nm^2/C^2)(-5 X 10^-^6)(3 X 10^-^6)/(0.2 m)^2
F_1_/_2 = -3.375 N


Q_1= -3.375 - F_1_3 = 7 N

- F_1_3= 7 - 3.375 = 3.625/-1= -3.65

F_1_3= -3.625

-3.625= (9 X 10^9 Nm^2/C^2)(3 X 10^-^6)(-8 X 10^-^6)/r^2

r^2 (-3.625)= (9 X 10^9 Nm^2/C^2)(3 X 10^-^6)(-8 X 10^-^6)

r^2= (9 X 10^9 Nm^2/C^2)(3 X 10^-^6)(-8 X 10^-^6)/(-3.625)
= \sqrt[2]{0.05958} r= 0.244 m

Cuestionario Equipo 3


Fuerza Eléctrica Superposición (Cuantitativa).


1.- Algunos de los electrones libres de un buen conductor (como un trozo de cobre, por ejemplo) se desplazan con una rapidez de 10^6 m/s ó más. Por qué estos electrones no escapan volando del conductor?

R: Pueden desarrollar esa velocidad cuando se aplica una diferencia de potencial entre los extremos de este (por ejemplo). Pero los electrones no podrían escapar de la red que conforma el sólido por las fuerzas interatómicas que los mantiene ligados, y solo se mueven los de las capas o niveles externos, y se alinean para dar lugar a una corriente eléctrica.

2.- Defina la aseveración siguiente: Si hubiese una sola partícula con carga eléctrica en todo el universo, el concepto de carga eléctrica carecería de significado?

R: El significado de carga eléctrica básicamente seria que se necesita otra partícula para generar la carga eléctrica que se representa por atracción y repulsión de partículas, entonces si no existiera otra partícula, no se generaría la carga eléctrica y por consecuente no aplicaría este significado.

3.- Dos cargas puntuales están situadas sobre el eje de las “y” como sigue: la carga Q1 = - 1.5 nC en y = - 0.6 m, y la carga Q2 = + 3.2 nC en el origen (y = 0). Cuál es la fuerza total (magnitud y dirección) que estas dos cargas ejercen sobre una tercera carga Q3 = + 5 nC que se encuentra en y = - 4 m?

R:

F_3_/_1 = (9 X 10^9 Nm^2/C^2)(5 X 10^-^9)(-1.5 X 10^-^9)/(3.4 m)^2
F_3_/_1 = (5.839 X 10^-^9)


F_3_/_2= (9 X 10^9 Nm^2/C^2)(5 X 10^-^9)(3.2 X 10^-^9)/ (4)^2


F_3_/_2 = 9 X 10^-^9


Q_3 = F_3_/_1 - F_3_/_2 = 5.8391 X 10^-^9 - 9 X 10^-^9

Q_3 = -3.1609 X 10^-^9 hacia abajo

4.- Dos cargas puntuales están situadas sobre el eje de las “x” como sigue: la carga Q1 = + 4 nC está en x = 0.2 m, y la carga Q2 = + 5 nC están en x = - 0.3 m. Cuáles son la magnitud y dirección de la fuerza total que estas dos cargas ejercen sobre una carga puntual negativa Q3 = - 6 nC que se encuentra en el origen?

R:

F_3_/_2 = (9 X 10^9 Nm^2/C^2)( 6 X 10^-^9)( 5 X 10^-^9)/(0.3 m)^2= 3X 10^-^6
F_3_/_1 = (9 X 10^9 Nm^2/C^2)(6 X 10^-^9) (4 X 10^-^9)/(0.2 m)^2= 5.4 X 10^-^6

Q_3= F_3_/_2 - F_3_/_1 = 3X 10^-^6 - 5.4 X 10^-^6
Q_3= 2.4 X 10^-^6 N hacia la derecha

Cuestionario Equipo 4

Campo Eléctrico: Carga Puntual.

1.- Se coloca un protón en un campo eléctrico uniforme y luego se libera. Después se coloca un electrón en el mismo punto y se libera. Experimentan estas dos partículas la misma fuerza?

R: Si, ya que el campo al que están sometidos es el mismo para ambos casos y por lo tanto están sometidos a la misma fuerza de este.

Y la misma aceleración?

R: No, ya que sus masas son diferentes y por lo tanto, aplicando la segunda Ley de Newton ( F= mg ), la aceleración depende de la masa y no solo de la fuerza que se aplique al protón y al electrón respectivamente.

Se desplazan en la misma dirección al ser liberadas?

R: No, ya que en caso de que el campo sea generado por una partícula negativa, al protón al liberarse se alejara de esta partícula (la repulsión), y el electrón en cambio se vera atraído por esta partícula (atracción), si el campo es generado por una partícula positiva las fuerzas serian de manera viceversa.

2.- Los campos eléctricos suficientemente intensos pueden provocar que los átomos se ionicen positivamente, esto es, que pierdan uno ó más electrones. Explicar como ocurre esto.

R: Los campos eléctricos generan niveles de energía, al absorber la energía un electrón se excita y pasa a un mayor nivel de energía, si el campo eléctrico produce ‘‘demasiada energía’’, esta será suficiente para que el electrón no solo pase a otro nivel de energía sino que tendera a saltar del átomo para formar un enlace con otro enlace.

Qué es lo que determina la intensidad que el campo debe tener para que esto ocurra?

R: El nivel de energía del electrón de valencia ( el que ocupa el ultimo nivel de energía dentro del átomo), en el que se encuentra este, ya que este nivel determinara que tan dispuesto esta el electrón para saltar del átomo o que tanta intensidad requiere el electrón para lograr la ionización.

3.- Cierta partícula tiene una carga – 3 nC. a) Hallar la magnitud y dirección del campo eléctrico debido a esta partícula en un punto situado 0.25 m directamente arriba de ella, b) A que distancia de esta partícula tiene su campo eléctrico una magnitud de 12 N/C?.

a) R:

E = (1/4\pi  \varepsilon_0)(q/r^2)(1/4\pi  \varepsilon_0)= 3 X 10^-^9/(0.250 m)^2 = 432 N/C

b) R:

E= 12 N/C = (1/4\pi  \varepsilon_0)(q/r^2) = \sqrt[2]{(1/4\pi \varepsilon_0)(3 X 10^-^9C)/12NC} = 1.50 m



4.- Un electrón inicialmente en reposo se deja libre en un campo eléctrico uniforme. El electrón se acelera verticalmente hacia arribar recorriendo 4.5 m en los primeros 3 us después de ser liberado.

a) Cuáles son la magnitud y dirección del campo eléctrico?

R:

X= 1/2 at^2

a= 2x/t^2= 2(4.50m)/3 X 10^-^6 s^2= 1 X 10^1^2 m/s^2

E= F/q= ma/q =( 9.11 X 10^-^3^1kg)(1 X 10^1^2 m/s^2)/1.6 X 10^-^1^9C

= 5.69 N/C

E = K q/r2

= 9 x 109 nm2/C2 (1.602 x 10 -19 C / (4.5 m)2

= 7.12 x 10 -11 n/c

b) Se justifica no tener en cuenta los efectos de la gravedad?, justificar la respuesta cuantitativamente.

Cuestionario Equipo 5

Campo Eléctrico: Debido a un Dipolo.

1.- La temperatura y velocidad del aire tiene valores diferentes en distintos lugares de la atmósfera terrestre. Es la velocidad del aire un campo vectorial?. Por que?. Es la temperatura del aire un campo vectorial? Por que?.

R: El campo vectorial es un conjunto infinito de cantidades que se encuentran asociadas con cada punto del espacio. Si decimos que la velocidad del aire es un campo vectorial estamos en lo incorrecto ya que la velocidad no se encuentra relacionada específicamente con un solo factor que se encuentre en el espacio. Ahora si decimos que la temperatura del aire es un campo vectorial estamos en lo correcto ya que la temperatura se encuentra relacionada con los factores en los que se manifiesta.

2.- Un objeto pequeño que tiene una carga de – 55 uC experimenta una fuerza hacia debajo de 6.2 x 10^9 N cuando se coloca en cierto punto de un campo eléctrico, a) Cuáles son la magnitud y dirección del campo eléctrico en este punto?, Cuáles serían la magnitud y dirección de la fuerza que actúa sobre un núcleo de cobre (número atómico = 29) masa atómica = 63.5 g/mol) situado en este mismo punto del campo eléctrico?

a) R:

q= -55 X 10^-^6 y F es hacia abajo con magnitud 6.20 X 10^-^9 N. por lo tanto,

E= F/q = 1.13 X 10^-^4 N/C, hacia arriba.

b) R:

si un núcleo de cobre se pone en ese punto, siente una fuerza ascendente de la magnitud


F= qE= (29)(1.6 X 10^-^1^9C)( 1.13 X 10^-^4 N/C) = 5.24 X 10^-^2^2 N

3.- En un sistema de coordenadas rectangulares se coloca una carga positiva puntual Q = 6x10^-9 C en el punto x = + 0.15 m, y = 0, y una carga puntual idéntica en x = - 0.15 m, y = 0. Hallar las componentes x y y, así como la magnitud y la dirección del campo eléctrico en los puntos siguientes: a) el origen; b) x = 0.3 m, y =0; c) x = 0.15 m, y = - 0.4 m; d) x = 0, y = 0.2 m.

a) R:


 5.24 X 10^-^2^4

 5.24 X 10^-^2^4

en el origen E= 0

b) R:

en x= 0.3 m y Y= 0 :

 5.24 X 10^-^2^4


\bar{E} = 1/4\pi  \varepsilon_0( 6 X 10^-^9 C) ( 1/(0.15 m)^2+ 1/(0.45 m)^2)\hat{i} = 2667\hat{i} N/C

c) R:

en X= o.15m y Y-0.4m

\bar{E} = 1/4\pi  \varepsilon_0( 6 X 10^-^9 C) = ( -1/(0.4 m)^2 \hat{j} + 1/(0.5 m)^2 \hat{i}- 1/ (0.5 m)^2(5)\hat{j})

\bar{E}=( 129.6\hat{i} - 510.3 \hat{j}) N/C= \bar{E}= 526.5 N/C y  \theta = 75.7 grados abajo de la X.

d) R:

X = 0 y Y= 0.2m

\bar{E} = 1/4\pi  \varepsilon_0 = 2(6 X 10^-^9)(0.2/0.25)/ (0.25 m)^2 = 1382\hat{j} N/C

Cuestionario Equipo 6

Campo Eléctrico: Problemas.

1.- Dos partículas con cargas Q1 = 0.5 nC y Q2 = 8 nC, están separadas por una distancia de 1.2 m. En qué punto a lo largo de la recta que une las dos cargas es igual a cero el campo eléctrico total debido a ambas cargas?

R:

Cargas de punto q1 (0.500 nC) y q2 ( 8.00 nC) se sepran cerca x= 1.20 m es cero cuando;

= = k/ = k /(1.20 - ) = = (1.2 - ) = - 2 (1.2) +

= () + 2(1.2) q1r1- = 0 ó 7.5 + 1.2 - 0.72 = 0

= 0.24, -0.4 = 0.24 se separan cerca


E=F/q = K q1/r2 = 9x109(Nm2/C2)(0.5x10-9C)/(0.2m)2 = 112.5 N/C

E=F/q = K q2/r2 Þ 112.5= K q1/x Þ x =K q2/112.5(N/C)= 0.8m

E= K q2/(0.8)2 = 112.5 N/C X= 0.8m

ET = q1 – q2 =112.5 N/C - 112.5 N/C = 0

ET = 0


2.- Una carga puntual de + 2 nC está en el origen, y una segunda carga puntual de – 5 nC está sobre el eje de las x en x = 8 m. a) Hallar el campo eléctrico (magnitud y dirección) en cada uno de los puntos siguientes sobre el eje de las x: i) x = 0.2 m; ii) x = 1.2 m; iii) x = - 0.2 m. b) Hallar la fuerza eléctrica neta que las dos cargas ejercerían sobre un electrón colocado en cada punto del inciso a).

a) R:

Carga de punto q_1(2.00 nC) esta en el origen y q_2(-5.00 nC) esta en X= 0.800 m


i) X= 0.200 m ; E= \left K|{q_1}\right | / (0.200 m)^2 + K\left |{q_2}\right | / (0.600 m)^2 = 575 N/C


ii) X= 1.20 m ; E= K\left |{q_1}\right | / (0.400 m)^2 + K\left |{q_1}\right | / ( 1.20 m )^2 = 269 N/C


iii) X= -0.200 m ; E= K\left |{q_1}\right | / (0.200 m)^2 + K\left |{q_2}\right | / (1.00 m)^2= 405 N/C




E=F/q = K q1/r2 carga 1 (+2nC) E1 carga 2 (-5nC)

x = 0.2 m; ii) 9x109(2x109)/(0.2m)2 = 450 N/C E= K (-5x10-9)/(0.2m)2= -1.12 N/C

x = 1.2 m; iii) 9x109(2x109)/(1.2m)2 = 12.5 N/C E= K (-5x10-9)/(1.2m)2= -312.5 N/C

x = - 0.2 m. 9x109(2x109)/(-0.2m)2 = 450 N/C E= K (-5x10-9)/(-0.2m)2= -1.125N/C


b) R:


F= -eE


i) F= 1.6 X 10^-^1^9 C · 575 N/C = 9.2 X 10^-^1^7 N


ii) F= 1.6 X 10^-^1^9 · 269 N/C = 4.3 X 10^-^1^7 N


iii) F = 1.6 X 10^-^1^9 · 405 N/C = 6.48 X 10^-^1^7 N



ET

E1 – E2= 450 N/C – (-1.12 N/C) = 451.12 N/C

E1 – E2= 12.5 N/C – (-312.5 N/C) = 325 N/C

E1 – E2= 450 N/C – (-1.125 N/C) = 451.12 N/C


Cuestionario Equipo 7

Flujo Eléctrico.

1.- Si se aumentan todas las dimensiones de la siguiente figura, por un factor de tres, Qué efecto tendrá este cambio en el flujo eléctrico a través de la caja?
Carga positiva adentro de la caja, flujo saliente.
R:

Ninguno, ya que la cantidad de líneas que atraviesen la caja serian las mismas sin importar las dimensiones de la caja, existiría un cambio en el flujo eléctrico si aumentáramos el valor de la carga ya que eta produciría una mayor cantidad de líneas que atravesaran la superficie de la caja.


2.- A fin de generar la cantidad máxima de energía eléctrica, los paneles solares se instalan de modo que estén aproximadamente de cara al Sol como sea posible. Explicar en qué sentido esta orientación es análoga a la obtención del flujo eléctrico máximo a través de una superficie plana.

R:

Esta orientación se relaciona con la relación en que entre mayor ángulo de captación del sol tengan los paneles solares, mayor será la cantidad de rayos recibidos por estos y mayor la cantidad de energía generada, esto es igual en relación al flujo eléctrico, entre mayor cantidad o mas líneas de fuerza atraviesen una superficie plana mayor será el valor del flujo eléctrico.


3.- Una hoja plana de papel con área de 0.25 m^2 está orientada de modo tal que la normal a la hoja forma un ángulo de 60º con un campo eléctrico cauniforme cuya magnitud es de 14 N/C, a) Hallar la magnitud del flujo eléctrico a través de la hoja; b) Depende de la respuesta del inciso a) de la forma de la hoja? Por que?; c) Con qué ángulo Φ entre la normal a la hoja y campo eléctrico es la magnitud del flujo a través de la hoja i) máxima? ii) mínima? Explicar las respuestas.

a) R:

A= 0.25 m2

θ= 60̊ = (14N/C)(cos60̊)(0.25 m2 )

=1.75 N m2 /C

E= 14 N/C

Φ= ¿?


b)R:

No, porque si se conoce el valor del área total, la respuesta no depende de la forma de la superficie.


c) R:

i) con un angulo totalmente perpendicular al flujo = 90̊ la magnitud del flujo es Maxima.

ii) con un angulo paralelo al flujo la magnitud del flujo es minima.



4.- Un cubo tiene lados de longitud L. Está colocado con un vértice en el origen como se muestra en la figura. El campo eléctrico es uniforme y está dado por E = - Bi, + Cj – Dk, donde B, C y D son constantes positivas. A) Hallar el flujo eléctrico a través de cada una de las seis caras del cubo S1, S2, S3, S4, S5, S6. b) Hallar el flujo eléctrico a través de todo el cubo.


Cuestionario Equipo 8

Ley de Gauss.

1.- Cuál es el flujo eléctrico total a través de una superficie que encierra totalmente un ion litio negativo? Cómo influiría en la respuesta el hecho de que se extendiera la superficie sin dejar de encerrar el ion (y ninguna otra carga)?

R:

 \varepsilon =( 9 X 10^9)    (1 X 10^-^9) / (0.60 X 10^-^1^0)mm^2


 \Phi= \epsilon  4\pi r^2 = 1.5 X 10 ^1^0^m^m^2 X 4\pi (0.60 X 10^-^1^0)mm^2 = - 6.7858 x 10^-^1^0 C

litio----radio ionico= (0.60 X 10^-^1^0)mm^2

2.- Se coloca una cantidad conocida de carga Q en el conductor de forma irregular que se muestra en la figura. Si se conoce el tamano y la forma del conductor, Se puede utilizar la ley de Gauss para calcular el campo eléctrico en una posición arbitraria externa al conductor?

R:

Si porque con la ley de gauss podemos calcular tanto el campo electrico interior de la particula como el exterior.

3.- Una superficie cerrada contiene una carga neta de -3.6 uC. Cuál es el flujo eléctrico neto a través de la superficie?, b) El flujo eléctrico a través de la superficie cerrada resulta ser de 780 N m^2/C, Qué cantidad de carga encierra la superficie?, c) La superficie cerrada del inciso b) es un cubo de con lados de 2.5 cm de longitud. Con base en la información dada en el inciso b), Es posible saber dónde está la carga dentro del cubo?. Explicar la respuesta.
R:

lado= 2.5 cm  \varepsilon = (9 X 10^9)(-3.5 X 10^-^9) / (0.0125 X 10^-^2m) = (-2.07 X 10^-^9C) vol= (2.5) = 0.0125m


 \Phi= \epsilon  4\pi r^2 = (-2.07 X 10^-^9C) X 4\pi (0.0125 X 10^-^2 m) = (-4.06 X 10^-^1^6)

Sacamos el radio del cubo dividiendo el valor de la arista entre 2, despues de eso podremos facilmente sacar el campo electrico aplicando la formula de la ley de gauss y una vez obteniendo ese resultado sacaremos el flujo electrico con la formula de  \Phi= \epsilon  4\pi r^2 y listo.

4.- En cierta región del espacio el campo eléctrico E a) es uniforme. Utilizar ley de Gauss y verificar que esta región de espacio debe ser eléctricamente neutra; es decir, la densidad volumétrica de carga ρ debe ser cero, b) Es cierta esta aseveración a la inversa; es decir, que en una región del espacio donde no hay carga E debe ser uniforme? Explicar la respuesta.

R:

Nos piden que saquemos el campo electrico y como no tenemos la carga pero tenemos el radio y el flujo, podemos usar la formula del flujo electrico para determinar el campo electrico y una vez determinado despejaremos la Q en la formula y obtendremos el resultado.

Cuestionario Equipo 9

Movimiento de una carga en un campo eléctrico: Introducción


1.- Una superficie gaussiana esférica encierra una carga puntual q. Si la carga puntual se des plaza del centro de la esfera a un punto alejado del centro, Cambia el campo eléctrico en un punto de la superficie? Cambia el flujo total a través de la superficie gaussiana? Explicar la respuesta.

2.- Una esfera metálica sólida con un radio de 0.45 m tiene una carga neta de 0.25 nC. Hallar la magnitud del campo eléctrico, a) En un punto situado a 0.1m afuera afuera de la superficie de la esfera; b) en un punto dentro de la esfera, a 0.1 m debajo de la superficie.
3.- En una demostración de clase de física se coloca una carga de - 0.18 uC en el domo esférico de un generador Van de Graaff; a) A que distancia del centro del domo se debe sentar usted para que el campo eléctrico en ese punto no exceda el máximo recomendado de 614 N/C (De acuerdo con las normas de seguridad del IEEE, Instituto de Ingenieros Electricistas y Electrónicos, los seres humanos deben evitar la exposición prolongada a campos eléctricos de magnitudes mayores que 614 N/C).

Cuestionario Equipo 10

Moviendo en un campo eléctrico: Problemas

1.- La ley de Coulomb y la ley de Gauss son totalmente equivalentes? Hay alguna situación de tipo electrostático en la que una sea válida y la otra no? Explicar el razonamiento.

2.- Cuántos electrones en exceso se deben agregar a un conductor esférico aislado de 32 cm de diámetro para producir un campo eléctrico de 1150 N/C inmediatamente afuera de su superficie?

3.- Una línea con carga uniforme y muy larga tiene una carga en cada unidad de longitud de 4.8 uC/m y yace a lo largo del eje de las x. Una segunda línea con carga uniforme y larga tiene una carga en cada unidad de longitud de
– 2.4 uC/m y es paralela al eje de las x en y = 0.4 m; Cuál es el campo eléctrico neto (magnitud y dirección) en los puntos siguientes del eje de las y: a) y = 0.2 m, b) y = 0.6 m?


Cuestionario Equipo 11

Potencial Eléctrico: Introducción Cualitativa.

1.- Cuál es la energía potencial total del siguiente sistema de tres cargas puntuales positivas, Q1 = Q2 = 2 uC que interactúan con una tercera carga Q3 = 4 uC. ¿Es positivo o negativo el resultado? ¿Cuál es la interpretación física de este signo?


2.- Si el potencial eléctrico en cierto punto es cero. ¿Debe ser igual a cero el campo eléctrico en ese punto? (Sugerencia considérese el campo de un dipolo eléctrico y el potencial de dos cargas puntuales)

3.- Una partícula pequeña tiene una carga de – 5 uC y una masa de 2 x 10^-4 Kg. Se traslada desde el punto A, donde el potencial eléctrico es Va = + 200 V, al punto B, donde el potencial eléctrico es Va = + 800 V. La fuerza eléctrica es la única fuerza que actúa sobre la partícula. Ésta tiene una rapidez de 5 m/s en el punto A. Cuál es su rapidez en el punto B? Se traslada con más rapidez o más lentamente en B que en A? Explicar la respuesta.

4.- La dirección de un campo eléctrico uniforme es hacia el este. El punto B está a 2 m al oeste del punto A, el punto C está a 2 m al este del punto A, y el punto D está 2 m al sur del A. Con respecto a cada punto, Es el potencial en ese punto mayor, menor o el mismo que en el punto A. Explicar el razonamiento en el que se fundamentan sus respuestas?

Cuestionario Equipo 12

Potencial Eléctrico Campo y Fuerza.

1.- Si el campo eléctrico en cierto punto es cero, Debe ser inevitable que el potencial eléctrico sea cero en ese punto? (Sugerencia: Considerar el efecto de un campo de un anillo con carga).

R: El potencial eléctrico en cierto punto no puede ser 0, ya que es el trabajo que debe realizar una fuerza eléctrica para mover una carga positiva Q desde el infinito hasta ese punto, por lo tanto es un número positivo, por lo tanto, en condiciones de campo eléctrico nulo el potencial asociado es constante.

2.- Cómo cambiarían los diagramas de la siguiente figura si se invirtiera el signo de cada carga?

R: Pues, no viene ninguna figura aquí, pero tomando en cuenta esta:

En esta figura tiene una q1= -5µC, y la q2 = 2 µC, podemos ver que el campo eléctrico que ejerce la carga 1, es mayor debido a la carga de -5, y tiene una atracción con la carga 2, por tener signos contrarios

Al cambiarle el signo a la carga, vemos que si tiene la misma intensidad, y las dos siguen teniendo atracción una con la otra, simplemente cambia el signo de la carga, ya que en este punto solo indica, invertir el signo, mas no la carga.

3.- Una carga eléctrica total de 3.5 nC está distribuida uniformemente en la superficie de una esfera metálica con un radio de 24 cm. Si el potencial es cero en un punto en el infinito, hallar el valor del potencial a las distancias siguientes del centro de la esfera: a) 48 cm; b) 24 cm; c) 12 cm.
R:
a) 48 cm

= (9x109 Nm2/C2)(3.5x10-9 C) / (.48 m)2 = 136.71 N/C

b) 24 cm

= (9x109Nm2/C2)(3.5x10-9C) / (.24 m)2 = 546.875 N/C

c) 12 cm

= (9x109Nm2/C2)(3.5x10-9C) / (.12 m)2 = 2187.5 N/C



Un anillo delgado con carga uniforme tiene un radio de 15 cm y una carga total de + 24 nC. Se coloca un electrón sobre el eje del anillo, a una distancia de 30 cm de su centro, obligándolo a permanecer en reposo sobre el eje del anillo. Después se deja libre el electrón; a) Describa el movimiento consecutivo del electrón, b) Hallar la rapidez del electrón cuando éste alcanza el centro del anillo.

Cuestionario Equipo 13

Potencial Eléctrico, Energía y Potencia.

1.- En cierta región del espacio el potencial está dado por V = A + Bx + Cy^3 + Dz^2, donde A, B, C y D son constantes. Cuál es el campo eléctrico en esta región?
R:
V=x+3cy2+2dz

2.- Dos placas metálicas paralelas grandes tienen cargas opuestas de igual magnitud. Las separan una distancia de 45 mm y la diferencia de potencial entre ellas es de 360 V, a) Cuál es la magnitud del campo eléctrico (se supone uniforme) en la región entre las placas?, b) Cuál es la magnitud de la fuerza que este campo ejerce sobre una partícula con una carga de + 2.4 nC?, c) Con base en los resultados del inciso b), calcular el trabajo realizado por el campo sobre la partícula cuando ésta se traslada de la placa de mayor a la de menor potencial; d) Compare el resultado del inciso c) con el cambio de energía potencial de la misma carga, calculado a partir del potencial eléctrico.

R:
V1-V2=360V

a) Cuál es la magnitud del campo eléctrico (se supone uniforme) en la región entre las placas?,

donde:


V1 - V2 es la diferencia de potencial

E es la Intensidad de campo en newton/culombio

r es la distancia en metros entre los puntos 1 y 2

entonces:

= E

8000= E



b) Cuál es la magnitud de la fuerza que este campo ejerce sobre una partícula con una carga de + 2.4 nC?,

F= Eq

F=8000(2.4X109)

F=1.92X1013


c) Con base en los resultados del inciso

Ep=q(VAB)

Ep=2.4X109(360V)

Ep=8.64X1011


c) Calcular el trabajo realizado por el campo sobre la partícula cuando ésta se traslada de la placa de mayor a la de menor potencial;

Ep=q(VAB)

Ep=2.4X109(360V)

Ep=8.64X1011


d) Compare el resultado del inciso

Ep=q(VAB)

Ep=2.4X109(360V)

Ep=8.64X1011


c) con el cambio de energía potencial de la misma carga, calculado a partir del potencial eléctrico.